Help me with this answer I don’t it

Help Me With This Answer I Dont It

Answers

Answer 1

Answer:

f(-2) = g(-2) this is the answer


Related Questions

m26 is (2x - 5) and m_8 is (x + 5)º
What is m 3?
12
9
3 4
56
78

Answers

Answer:

12

Step-by-step explanation:

12 is the coefficient

Pls help I really need help beg you you

Answers

The correct answer is (0, 5)

Answer:0,5

Step-by-step explanation:

Choose the conditional statement that can be used with its converse to form the following biconditional statement: "It is a leap year if and only if the year has 366 days."
A. If it is not a leap year, then it does not have 366 days.
B. If it is a leap year, then the year has 366 days.
C. If a leap year has 366 days, then this is a leap year. D. If a year does not have 366 days, then it is not a leap year​

Answers

Given the biconditional statement: "It is a leap year if and only if the year has 366 days.", the converse to form it is "If the year has 366 days, then this is a leap year". (Right choice: C)

How to determine the propositional form of a sentence

According to logics, propostions are truth bearers that makes sentences true or false. In linguistics, propositions are the meaning of declarative sentences. There are simple and composite propositions, the latter are formed by one simple proposition at least and logic connectors. There are five logic connectors:

Conjuction     X ∧ Y   ("and" operator)Disjunction    X ∨ Y   ("or" operator)Negation       ¬ X       ("not" operator)Implication/Conditional          X ⇒ Y      ("if-then" operator)Double implication/Biconditional       X ⇔ Y     ("if-only if" operator)

By logic rules we know that the double implication/biconditional is commutative operator:

(X ⇔ Y) ⇔ (Y ⇔ X)

In addition, a double implication/biconditional has the following equivalence:

(X ⇒ Y) ∧ (Y ⇒ X)

Where Y ⇒ X is the converse of X ⇒ Y.

Therefore, the converse to form the statement "It is a leap year if and only if the year has 366 days" is Y ⇒ X: "If the year has 366 days, then this is a leap year".

To learn more on propositions: https://brainly.com/question/14789062

#SPJ1

In a random sample of 150 customers of a high-speed internet provider, 63 said that their service had been interrupted one or more times in the past month. Find a 95% confidence interval for the proportion of customers whose service was interrupted one or more times in the past month.

Answers

Answer:

The correct answer is "0.3410, 0.4990".

Step-by-step explanation:

Given values are:

[tex]n=150[/tex]

[tex]p=\frac{63}{150}[/tex]

  [tex]=0.42[/tex]

At 95% confidence interval,

C = 95%

z = 1.96

As we know,

⇒ [tex]E=z\sqrt{\frac{p(1-p)}{n} }[/tex]

By substituting the values, we get

       [tex]=1.96\sqrt{\frac{0.42\times 0.58}{150} }[/tex]

       [tex]=1.96\sqrt{\frac{0.2436}{150} }[/tex]

       [tex]=0.0790[/tex]

hence,

The confidence interval will be:

= [tex]p \pm E[/tex]

= [tex]0.42 \pm 0.079[/tex]

= [tex](0.3410,0.4990)[/tex]

Chung has 6 trucks and 5 cars in his toy box. Brian has 4 trucks and 5 cars in his toy box.

Which is the correct comparison of their ratios of trucks to cars?
StartFraction 6 Over 4 EndFraction less-than StartFraction 5 Over 5 EndFraction
StartFraction 6 Over 4 EndFraction greater-than StartFraction 5 Over 5 EndFraction
StartFraction 6 Over 5 EndFraction less-than StartFraction 4 Over 5 EndFraction
StartFraction 6 Over 5 EndFraction greater-than StartFraction 4 Over 5 EndFraction

Answers

Given:

Chung has 6 trucks and 5 cars in his toy box.

Brian has 4 trucks and 5 cars in his toy box.

To find:

The correct comparison of their ratios of trucks to cars.

Solution:

The ratio of trucks to cars is defined as:

[tex]\text{Ratio}=\dfrac{\text{Number of trucks}}{\text{Number of cars}}[/tex]

Chung has 6 trucks and 5 cars in his toy box. So, the ratio of trucks to cars is:

[tex]\text{Ratio}=\dfrac{6}{5}[/tex]

Brian has 4 trucks and 5 cars in his toy box.

[tex]\text{Ratio}=\dfrac{4}{5}[/tex]

We know that,

[tex]6>4[/tex]

[tex]\dfrac{6}{5}>\dfrac{4}{5}[/tex]

Therefore, the correct option is D.

Answer:

what the guy above me said

Step-by-step explanation:

so yeah he is right points

PLS HELP! I NEED TO FIND THE SURFACE AREA OF THIS CYLINDER!

PLS PROVIDE A STEP BY STEP EXPLANATION! ❤️

Answers

..........................................

Y= abx that goes through points (0,13) and (2,325)

Answers

Answer:

[tex]y = 13(5)^x[/tex]

Step-by-step explanation:

We are given the following exponential function:

[tex]y = ab^x[/tex]

(0,13)

This means that when [tex]x = 0, y = 13[/tex]. So

[tex]y = ab^x[/tex]

[tex]13 = ab^0[/tex]

[tex]a = 13[/tex]

Then

[tex]y = 13b^x[/tex]

(2,325)

This means that when [tex]x = 2, y = 325[/tex] We use this to find b. So

[tex]y = 13b^x[/tex]

[tex]13b^2 = 325[/tex]

[tex]b^2 = \frac{325}{13}[/tex]

[tex]b^2 = 25[/tex]

[tex]b = 5[/tex]

Thus

[tex]y = 13(5)^x[/tex]

Solve (2x-1)^2=8 using the quadratic formula

Answers

Step-by-step explanation:

The given equation is :

[tex](2x-1)^2=8[/tex]

or

[tex](2x-1)=\sqrt{8}\\\\2x-1=\pm 2\sqrt2\\\\2x=\pm 2\sqrt2 +1\\\\x=\dfrac{2\sqrt2 +1}{2}\\\\x=\pm (\sqrt 2+\dfrac{1}{2})\\\\or\\\\x=\sqrt2+\dfrac{1}{2},-\sqrt2-\dfrac{1}{2}[/tex]

Hence, this is the required solution.

This question is mildy difficult and i need some help would anyone please help me

Answers

Answer:

30 and 45 degrees

Step-by-step explanation:

Trình bày phương pháp tìm ma trận nghịch đảo bằng phép biến đổi sơ cấp. Áp dụng tìm ma trận nghịch đảo của ma trận

Answers

Answer:

bukhayung saging my faborito

Which equation represents a line that passes through (5, 1) and has a slope of ?
O y-5 = {(x-1)
Oy- } = 5(x –1)
Oy-1 = {(x–5)
Oy - 1 = 5(x-)

Answers

Answer:

y - 1 = 5(x - 5)

Step-by-step explanation:

Given the following data;

Points (x, y) = (5, 1)

Slope = ?

From the question, the value of the slope is missing. Hence, let's assume a value of 5.

Mathematically, the equation of a straight line is given by the formula;

y = mx + c

Where;

m is the slope.

x and y are the points

c is the intercept.

To find the equation of line, we would use the following formula;

y - y1 = m(x - x1)

Substituting into the formula, we have;

y - 1 = 5(x - 5)

y - 1 = 5x - 25

y = 5x - 25 + 1

y = 5x - 24 = mx + c

Solve the simultaneous equations.
Show all your working.
3x + 4y = 14
5x+2y=21

Answers

Step-by-step explanation:

hear is your answer in attachment

Does crime pay? The FBI Standard Survey of Crimes showed that for about 80% of all property crimes (burglary, larceny, car theft, etc.), the criminals are never found and the case is never solved.† Suppose a neighborhood district in a large city suffers repeated property crimes, not always perpetuated by the same criminals. The police are investigating nine property crime cases in this district.

Answers

Answer:

The appropriate answer is "11 crimes".

Step-by-step explanation:

Let,

The number of crimes be "n".

Now,

⇒ [tex]P(solved \ at \ least \ one) = 1-P(solved \ none \ of \ n)[/tex]

or,

⇒ [tex]1-(0.8)^n>0.9[/tex]

⇒       [tex](0.8)^n<0.1[/tex]

By taking log both sides, we get

⇒ [tex]n>=\frac{log(0.1)}{log(0.8)}[/tex]

By putting the values, we get

⇒ [tex]n>=10.31[/tex]

      [tex]n=11 \ crimes[/tex]

Answer these questions about the right triangle.

What is the area of the square of the leg 6?

What is the area of the square of the leg 8?

What is the area of the hypotenuse square?

What is the length of the hypotenuse?

Answers

9514 1404 393

Answer:

366410010

Step-by-step explanation:

The area of a square is the square of the side length. Then the side length is the square root of the area.

6² = 36 . . . area of the square of leg 68² = 64 . . . area of the square of leg 836+64 = 100 . . . sum of the two leg squares = area of hypotenuse square√100 = 10 . . . hypotenuse length

can someone help asap!! ​

Answers

7/9÷4/9=7/4

= 1/3/4

Hence, a=1, b=3, c=4.

hope it helps:)))

một nhà máy có 3 phân xưởng, tỉ lệ là 20%, 30%, 50%. tỉ lệ sản phẩm loại một của các phân xưởng lần lượt là 90%, 85%, 80%
lấy ngẫu nhiên 1 sản phẩm của nhà máy, tính xác suất để được sản phẩm loại một

Answers

Answer:

83.5%

Step-by-step explanation:

90% × 20% + 85% × 30% + 80%× 50%

WILL GIVE BRAINLIEST TO WHOEVER ANSWERS FIRST

Answers

Answer:

height = (3x + 3)

Step-by-step explanation:

area of rectangular prism = length *breadth

2x^2 + 3x = length*breadth

volume of a rectangular prism = length * breadth *height

6x^3 + 13x^2 + 6x = (2x^2 + 3x) *height

x(6x^2 + 13x + 6) = x(2x + 3) *height

a nd x gets cancel . So,

(6x^2 + 13x + 6) = (2x + 3) *height

6x^2 + (9 + 4)x + 6 = (2x + 3) *height

6x^2 + 9x + 4x + 6 = (2x + 3) *height

3x(2x +3) +2(2x +3) = (2x + 3) *height

(3x + 2)(2x + 3) = (2x + 3)*height

(3x + 2)(2x + 3) / (2x + 3) = height

(2x + 3 ) of numerator and (2x + 3) of denominator gets cancel

(3x + 3) / 1 = height

(3x + 3) = height

Use roster notation to name the set of all letters in the word ​"​more".
The set of all letters in the word ​"more​" is { }

Answers

9514 1404 393

Answer:

  {e, m, o, r}

Step-by-step explanation:

Perhaps most straightforward is the simple list in order of appearance.

  {m, o, r, e}

Often, we prefer rostered sets to be in lexicographical order:

  {e, m, o, r}

Business Weekly conducted a survey of graduates from 30 top MBA programs. On the basis of the survey, assume the mean annual salary for graduates 10 years after graduation is 181000 dollars. Assume the standard deviation is 31000 dollars. Suppose you take a simple random sample of 60 graduates.

Find the probability that a single randomly selected policy has a mean value between 172595.6 and 196608.1 dollars.
P(172595.6 < X < 196608.1) =
(Enter your answers as numbers accurate to 4 decimal places.)

Find the probability that a random sample of size
n
=
60
has a mean value between 172595.6 and 196608.1 dollars.
P(172595.6 < M < 196608.1) =
(Enter your answers as numbers accurate to 4 decimal places.)

Answers

Answer:

0.2979 = 29.79  probability that a single randomly selected policy has a mean value between 172595.6 and 196608.1 dollars.

0.982 = 98.2% probability that a random sample of size 60 has a mean value between 172595.6 and 196608.1 dollars.

Step-by-step explanation:

To solve this question, we need to understand the normal probability distribution and the central limit theorem.

Normal Probability Distribution:

Problems of normal distributions can be solved using the z-score formula.

In a set with mean [tex]\mu[/tex] and standard deviation [tex]\sigma[/tex], the z-score of a measure X is given by:

[tex]Z = \frac{X - \mu}{\sigma}[/tex]

The Z-score measures how many standard deviations the measure is from the mean. After finding the Z-score, we look at the z-score table and find the p-value associated with this z-score. This p-value is the probability that the value of the measure is smaller than X, that is, the percentile of X. Subtracting 1 by the p-value, we get the probability that the value of the measure is greater than X.

Central Limit Theorem

The Central Limit Theorem establishes that, for a normally distributed random variable X, with mean [tex]\mu[/tex] and standard deviation [tex]\sigma[/tex], the sampling distribution of the sample means with size n can be approximated to a normal distribution with mean [tex]\mu[/tex] and standard deviation [tex]s = \frac{\sigma}{\sqrt{n}}[/tex].

For a skewed variable, the Central Limit Theorem can also be applied, as long as n is at least 30.

On the basis of the survey, assume the mean annual salary for graduates 10 years after graduation is 181000 dollars. Assume the standard deviation is 31000 dollars.

This means that [tex]\mu = 181000, \sigma = 31000[/tex]

Find the probability that a single randomly selected policy has a mean value between 172595.6 and 196608.1 dollars.

This is the p-value of Z when X = 196608.1 subtracted by the p-value of Z when X = 172595.6. So

X = 196608.1

[tex]Z = \frac{X - \mu}{\sigma}[/tex]

[tex]Z = \frac{196608.1 - 181000}{31000}[/tex]

[tex]Z = 0.5[/tex]

[tex]Z = 0.5[/tex] has a p-value of 0.6915

X = 172595.6

[tex]Z = \frac{X - \mu}{\sigma}[/tex]

[tex]Z = \frac{172595.6 - 181000}{31000}[/tex]

[tex]Z = -0.27[/tex]

[tex]Z = -0.27[/tex] has a p-value of 0.3936

0.6915 - 0.3936 = 0.2979

0.2979 = 29.79  probability that a single randomly selected policy has a mean value between 172595.6 and 196608.1 dollars.

Sample of 60:

This means that [tex]n = 60, s = \frac{31000}{\sqrt{60}}[/tex]

Now, the probability is given by:

X = 196608.1

[tex]Z = \frac{X - \mu}{\sigma}[/tex]

By the Central Limit Theorem

[tex]Z = \frac{X - \mu}{s}[/tex]

[tex]Z = \frac{196608.1 - 181000}{\frac{31000}{\sqrt{60}}}[/tex]

[tex]Z = 3.9[/tex]

[tex]Z = 3.9[/tex] has a p-value of 0.9999

X = 172595.6

[tex]Z = \frac{X - \mu}{\sigma}[/tex]

[tex]Z = \frac{172595.6 - 181000}{\frac{31000}{\sqrt{60}}}[/tex]

[tex]Z = -2.1[/tex]

[tex]Z = -2.1[/tex] has a p-value of 0.0179

0.9999 - 0.0179 = 0.982

0.982 = 98.2% probability that a random sample of size 60 has a mean value between 172595.6 and 196608.1 dollars.

If you become a. . what Will you do

Answers

Answer:

if you becomes a.. what is this mean

Answer:

i have no idea

Step-by-step explanation:

i dont think turning into a virtual period will let me do anything. If you can clarify please let us know!

A farmer makes a rectangular enclosure for his animals.
He uses a wall for one side and a total of 72 metres of fencing for the other three sides.
The enclosure has width x metres and area A square metres.
Show that A = 72x - 21.

Answers

Answer:

Remember that, for a rectangle of length L and width W, the area is:

A  =L*W

And the perimeter is:

P = 2*(L + W)

In this case, we know that:

W = x

Let's assume that one of the "length" sides is on the part where the farmer uses the wall.

Then the farmer has 72 m of fencing for the other "length" side and for the 2 wide sides, then:

72m = L + 2*x

isolating L we get:

L = (2x - 72m)

Then we can write the area of the rectangle as:

A = L*x = (2x - 72m)*x

A = 2*x^2 - 72m*x

(you wrote  A = 72x - 21, I assume that it is incorrect, as the area should be a quadratic equation of x)

Christen returned an overdue book to the library. Her fine was the same as the amount on the following day. Out of the fines shown below, which one would be closest to the number of days overdue?

Answers

Answer: 21 days overdue

Explanation: Since her fine was same as the amount on the following day. 21 will be closest to the number of day overdue so it is 21 days.

Answer:

22

Explanation:

 They are saying that it has to be the closest number to the number of days overdue, cannot be neither 10 nor 14, and it cannot be 18 because that's one day from the days it's been overdue, and the question is saying: Her fine was the same as the amount on the following day. The graph shows us that it is about 1 more day than 18, so the only reasonable answer here is 22.

Also, I got it correct on edg =)

Simplify the expression

Answers

Answer:

6

Step-by-step explanation:

3 sqrt(20) / sqrt(5)

We know that sqrt(a) /sqrt(b) = sqrt(a/b)

3 sqrt(20/5)

3 sqrt(4)

3 *2

6

what the person above me said

Is AABC-ADEP? If so, identify the similarity postulate or theorem that
applies.
B
16
105°
5-105
4
36
A
C
D
9
F
I

Answers

Answer:

D. cannot be determined

Step-by-step explanation:

that is kind of a trick question.

right based on the given information none of the 3 methods can be applied.

we don't have all 3 sides for SSS

we don't have 2 angles for AA

and we don't have 2 sides with their included angle for SAS.

but it can still be determined (they ARE similar), as this is a special case of SSA, where it is clear that the corresponding congruent angles are located at the same position relatively to the corresponding similar lines.

you can discuss this with your teacher, if you are interested.

AND - we can easily determine any of the other missing pieces (side and angles). these missing pieces then will correspond exactly to their counterparts in the other triangle (the 2 angles in ABC are equal to the 2 angles in DEF, the missing sides file the same scaling factor as the other known sides). and then any of the 3 methods can be applied.

so, this is actually tricky ...

What is the answer plz help!!!

Answers

9514 1404 393

Answer:

  B.  horizontal stretch by a factor of 3

Step-by-step explanation:

Replacing x by x/k in a function causes its graph to be stretched horizontally by a factor of k. (This makes sense because it means x needs to be k times as large to give the same argument to the function.)

Here, the value of k is 3, so the function graph is horizontally stretched by a factor of 3.

The sum of two numbers is 12 The product of the smaller
umber and 3 is -6. Find
the numbers​

Answers

Answer:

-2 and 14

Step-by-step explanation:

-2×3=-6-2+14=12

The numbers are -2 and 14

What is algebraic expression?

A number, a variable, or a mix of numbers, variables, and operation symbols make up an expression. Two expressions joined by an equal sign form an equation. Word illustration: The product of 8 and 3. Word illustration: The product of 8 and 3 is 11

given

Let smaller number be x and larger number be y

x * 3 = -6

x = -6/3 = -2

x+y = 12

-2 + y = 12

y = 12+2 = 14

to learn more about algebraic expression refer to:

https://brainly.com/question/4344214

#SPJ2

find the interior angle of a regular polygon with 20 sides. What is the size of exterior angle​

Answers

Answer: 18

Step-by-step explanation:

360/20 = 18

Find the volume of a sphere with a surface area of 400 square feet.

Answers

Answer:

V≈752.25ft

Step-by-step explanation:

please mark me brain list

Answer: the volume of the sphere would be precicely be 752.25278

Step-by-step explanation: yeah, i guess this is just a run and search kind of question, the formula to find it was:

A=4πr2

V=4

3πr3

Solving forV

V=A3/2

6π=4003/2

6·π≈752.25278

There are 17 books on a shelf. 8 of these books are new. the rest of them are used. (GIVING BRAINLEST TO BEST ANSWER) what is the ratio?​

Answers

A. 9 to 8
B. 17 to 9?

These marbles are placed in a bag and two
of them are randomly drawn.
What is the probability of drawing two pink
marbles if the first one is placed back in the
bag before the second draw?
Give your answer as a ratio, reduced to
simplest terms.
[?]
Hint: Multiply the probability of the 1st Event by the
probability of the 2nd Event to get your answer.
Enter

Answers

Answer:  9/100

========================================================

Explanation:

There are 3 pink marbles out of 10 total (we can see that 2 yellow + 3 pink + 5 blue = 10 total)

The fraction 3/10 represents the probability of pulling out a pink marble at random. Since the first one is put back, this means the fraction 3/10 stays the same on the second draw as well.

We use the idea mentioned in the hint to multiply the fractions:

(3/10)*(3/10) = (3*3)/(10*10) = 9/100

The probability of pulling two pink marbles in a row is 9/100, where the first marble is put back.

Effectively, if you were given 100 tries at doing this, you should expect about 9 of those times you get 2 pink marbles in a row.

Other Questions
The city of williamsburg decided to defease old 6% bonds carried in its electric enterprise fund with new 4.5% bonds. As a result of the defeasance, the city incurred an accounting loss. This loss should be recognized:_______a. As an adjustment to retained earnings since it is applicable to prior periods. b. In the year of the defeasance. c. Over the remaining life of the old bonds or the new bonds whichever is shorter d. It should not be recognized Zoey Bella Company has a payroll of $10,000 for a five-day workweek. Its employees are paid each Friday for the five-day workweek. Journalize the adjusting entry required on December 31, assuming the year ends on a Thursday. If an amount box does not require an entry, leave it blank.DateDescriptionPost. Ref.DebitCredit How much will this trash can hold? It is 4 feet tall and has a base radius of 1 foot._____ cubic feet Cc enzyme xc tc phn ng thng c cu trc bc my? Ti sao? Disuss the era before Greek and Roman historiography(proto-history) define reaction rate The most important belief in lslam are the? Find the set of values of k for which the line y=kx-4 intersects the curve y=x-2x at 2 distinct points? Kelley Company reports $960,000 of net income and declares $120,000 of cash dividends on its preferred stock for the year. At year-end, the company had 400,000 weighted-average shares of common stock. 1. What amount of net income is available to common stockholders? y>2x y>3 orderd pair Natasha and Link have been married for 2 years. They live in North Carolina and are about to make an offer on their first home. Their goal is to own the property so that if either Natasha or Link were to die, the surviving spouse would own the property outright. They also want to keep things private and avoid probate. How should they title their new home?A. Link should own the house fee simple and name Natasha as his beneficiary. B. Tenancy in common is the best way to title the property because it meets all their objectives. C. They should title the home as JTWROS as a way to accomplish their goals. D. Because they live in North Carolina, they should title the home as community property. What did Mr. Logan do for a living? PLease Help!! will mark brainilest!! What is anequation of the line that passes through the points (0, -1) and (6, 1)? If a number is chosen at random from the numbers 1 to20 inclusive, what is the probability that a single-digitnumber will be picked? ( give your answer in simplifiedfractions eg. 2/5 when necessary) Which of the following best describes the use of the formula S = (n - 2)180,where n is the number of sides?A. It is used to find the number of interior angles in a regular polygon.B. It is used to find the sum of the interior angles in a regularpolygon.O c. It is used to find the sum of the exterior angles in a regularpolygonO D. It is used to find the number of exterior angles in a regularpolygonSUBMIT Identify the possible error in each line. For each line assume that the lines above it are part of the same code and have been fixed by the time you get to the line you are currently examining. . Yes there are some lines with No Errors - NONE is not a red herring. a) int intA = 10; NONE b) int& iptrA = *intA; SYNTAX int *arrA = (int) malloc("iptrA); SEMANTIC for(int i = 0; i < intA; d) i++) NONE e) arrA[ I ]=i; NONE f) for(int i = 1; i < intA; i++) [Select] { cout >>"index ::" g) >> >> "="; Select] 4 cout Which best illustrates the way in which radiation transfers thermal energy?OWarrCooloWarmCoolWarmCoolWarmH11Cool 3. You are working part time providing computer support for a veterinarians office. When you arrive to work one morning, the receptionist informs you that the computer monitor is not displaying anything. List at least three steps you will perform to troubleshoot the problem, and list three possible causes. Aztec society was Group of answer choices an egalitarian democracy. primarily involved in the trading of slaves. a purely theocratic society. a hierarchical dictatorship, with a privileged upper class and a downtrodden majority. unique in giving women major political power.